2013 AMC 10B Problems/Problem 1

Revision as of 22:04, 13 February 2017 by Mathguy5041 (talk | contribs) (Solution)

Problem

What is $\frac{2+4+6}{1+3+5} - \frac{1+3+5}{2+4+6}$?

$\textbf{(A)}\ -1 \qquad\textbf{(B)}\ \frac{5}{36}  \qquad\textbf{(C)}\ \frac{7}{12} \qquad\textbf{(D)}\ \frac{49}{20} \qquad\textbf{(E)}\ \frac{43}{3}$

Solution

This expression is equivalent to $\frac{12}{9} - \frac{9}{12} = \frac{16}{12} - \frac{9}{12} = \boxed{\textbf{(C) }\frac{7}{12}}$

For the exact same problem but with answer choice D unsimplified, see http://artofproblemsolving.com/wiki/index.php/2013_AMC_10B_Problems/Problem_1

See Also

2013 AMC 10B (ProblemsAnswer KeyResources)
Preceded by
First Problem
Followed by
Problem 2
1 2 3 4 5 6 7 8 9 10 11 12 13 14 15 16 17 18 19 20 21 22 23 24 25
All AMC 10 Problems and Solutions

The problems on this page are copyrighted by the Mathematical Association of America's American Mathematics Competitions. AMC logo.png